If Sukanya receives six e–mail messages, the fifth of which is from Lula, which one of the following must be true?

Harper on July 7, 2020

Confused

First, I would like to note that in another comment on this question, Mehran says there will be a video for this game soon. 7 years later and still no video. But my question is. Isn’t it possible to have: H 1,7 J 2,3 L 4,5 And yet this is a must be true question, can you please explain this apparent contradiction, maybe I am overlooking something. Thank you in advance

Replies
Create a free account to read and take part in forum discussions.

Already have an account? log in

shunhe on July 8, 2020

Hi @Harper,

Thanks for the question! So actually, HJJLLH isn’t a possible combination. How do we know that? Well, it violates the last rule, that of the first three message, EXACTLY one is from J. And HJJLLH has 2 messages from J in the first three messages, which contradicts this last rule.

So putting L in fifth, what can we put in 1 and 6? Obviously not L, though H does work (as you seemed to figure out). J doesn’t work because we need to have one HJ block. So H has to be first and sixth.

Then we also know that J has to be 2nd. Because of the exactly one J in the first three rule, the other J has to go in 4, and the second L has to go in three. So when L is in fifth out of six messages, there’s only one possible scenario: HJLJLH. And then we can easily see that the answer is (D).

Thanks for bringing the video to our attention. Please direct any support related issues (including video explanations) to our support staff by tapping "support" from the left menu or by calling 855.483.7862 ext. 2 Monday-Friday 9am-6pm PT. Sorry about that!

Hope this helps! Feel free to ask any other questions that you might have.

Harper on July 16, 2020

Thank you this actually helped me out a lot. When I wrote down my rules I put that J has to appear in first 3 but forgot the part that it appears only once. Thanks again!